Hitungkan pecutan bagi blok di bawah: / Cal
(a)
m= 2 kg
F= 8.0 N​

Answers

Answer 1

Answer:

Acceleration = 4 m/s²

Explanation:

Given the following data;

Force = 8 N

Mass = 2 kg

To find the acceleration of the block;

Newton's Second Law of Motion states that the acceleration of a physical object is directly proportional to the net force acting on the physical object and inversely proportional to its mass.

Mathematically, it is given by the formula;

[tex] Acceleration = \frac {Net \; force}{mass} [/tex]

Substituting into the formula, we have;

[tex] Acceleration = \frac {8}{2} [/tex]

Acceleration = 4 m/s²


Related Questions

An object is sitting on the floor. A 22.4 N force is pulling the object to the right and an 11 N force is pulling the object to the left. The gravitational force on the object is 70 N. What is the net force?

Answers

answer = 33.4 net force.

A force of 20000N acts on the raft in the direction down
State the name given to the force shown by arrow in Fig.
Calculate the mass of the raft.

Answers

Answer:

Figure is not there

Explanation:

HELPPP THIS IS IMPORTANT NOWW OMGG HELPPPP PLEASEE

Answers

Answer:

The correct answer is -

19 - C) parasitism

20- B) commensalism

Explanation:

Parasitism is a type of symbiotic relationship among two different organisms where one is benefited and the other is harmed. The harmed organism known as the host and the benefitted organism called the parasite. For instance, tapeworms are parasites in the human body.

Commensalism is also a symbiotic relationship but in this relationship, one symbiont is benefitted but another one neither benefitted nor harmed. For instance, Hermit crabs use the shells of dead snails for homes.

If a charge of 40kC flows through an electrical heater and the amount of energy converted into heat is 10MJ. Calculate the potential difference across the ends of the heater.

Answers

Answer:

Energy Transfer Quiz - Quiz

The diagram shows a heater above a thermometer. The thermometer bulb is in the position shown.

Kinetic and potential energy are related
a.) indirectly
b.) inversely
c.) directly
d.) Both A and B

Answers

Mark Brainliest please


Answer: B

The kinetic energy is inversely proportional to the potential energy.

potential energy is position relative, and kinetic energy is motion relative.
The primary relationship between the two is their ability to transform into each other. In other words, potential energy transforms into kinetic energy, and kinetic energy converts into potential energy, and then back again. It’s a never-ending cycle.

At the local grocery store, you push a 14.5-kg shopping cart. You stop for a moment to add a bag of dog food to your cart. With a force of 12.0 N you now accelerate the cart from rest through a distance of 2.29 m in 3.00 s. What was the mass of the dog food?

Answers

Answer:

The mass of the dog food added is 9.03 kg

Explanation:

Given;

mass of the shopping cart, m₁ = 14.5 kg

let the mass of the bag added = m₂

the force applied, F = 12 N

initial velocity of the cart-bag system, u = 0

distance traveled by the system, d = 2.29 m

time of motion of the system, t = 3.0 s

The acceleration of the system is calculated as;

[tex]d = ut + \frac{1}{2} at^2\\\\2.29 = 0 + (\frac{1}{2} \times 3^2)a\\\\2.29 = 4.5 a\\\\a = \frac{2.29}{4.5} \\\\a = 0.51 \ m/s^2[/tex]

The total mass of the system (M) is calculated as follows;

F = Ma

M = F/a

M = (12)/(0.51)

M = 23.53 kg

The mass of the dog food added is calculated as;

m₂ = M - m₁

m₂ = 23.53 kg - 14.5 kg

m₂ = 9.03 kg

calculate:An elcetric lamp is rated 240V and 40W. What is the cost of running the lamp for 62 hours if the elctricity authorities charges #2.50k per KWH

Answers

Answer:

Total cost = #6.2

Explanation:

Given the following data;

Voltage = 240 V

Power = 40 W

Time = 62 hours

Cost = #2.50

To find the cost of running the lamp for 62 hours;

First of all, we would determine the energy consumption of the electric lamp

Energy consumption = power * time

Energy consumption = 40 * 62

Energy consumption = 2480 Watt-hour = 2.48 Kwh (1 Kilowatts is equal to 1000 watts).

Next, we would determine the cost of running the lamp;

Total cost = energy consumption * cost

Total cost = 2.48 * 2.50

Total cost = #6.2


If a girl is running along a straight road with a uniform velocity 1.5 m/s, find her
acceleration in numerical type

Answers

Answer:

Explanation:

The definition of acceleration is the change in velocity over a period of time. If the girl's velocity is constant, that means it's not changing. Therefore, acceleration is 0 m/s/s

Helpppp!!!
You are sitting on a dock watching water waves as they pass. you noticed at 5 waves to pass by certain spot every 20 seconds you also noticed at the crest of the water line up exactly with two points that are 6 m apart.

A) what is the frequency of these waves?
B)what is the wavelength of the waves described above?
C) what is the velocity of the waves from where u are watching?
Please show ur workkkkk

Answers

Answer:

A) 0.25 Hz

B) 6 m

C) 1.5 m/s

Explanation:

The number of waves that pass a certain spot every 20 seconds = 5 waves

The distance between the crest = 6 m

A) The frequency of the wave, f = The number of cycles per second = 5/20 = 0.25 per second = 0.25 Hz

B) The wavelength of the wave, λ = The distance between crests = 6 m

C) The velocity of the wave, v = f × λ = 0.25/s × 6 m = 1.5 m/s.

1. The mechanical energy of the pendulum at the highest point equals
a. kinetic energy only.
b. kinetic energy x potential energy.
c. potential energy only.
d. zero.

Answers

Answer:

c. potential energy only

Explanation:

at highest point the body will having some height and gravitational force will work on it.

since the velocity at highest point will be zero therefore there will no kinetic energy

A student starts at position (4,4) m and undergoes three displacements:
d1 = (2, -3) m, d2 = (-5, 0) m, and d3 = (1, 6) m.
What is the final position of the student? Show your solution graphically and numerically.
please help!!

Answers

Answer:

The final position is  (2, 7)m

Explanation:

When we work with coordinate pairs, the addition works as:

(a, b) + (c, d) = (a + c, b + d)

So, for example, if we start at (a, b), and we have a displacement d = (1, 1)

we just need to solve:

(a, b) + (1, 1) = (a + 1, b + 1)

Now, in this case, we start at (4, 4)m

first, we have d1 = (2, -3) m

After this displacement, the position is:

(4, 4)m + (2, -3)m = (4 + 2, 4 - 3)m = (6, 1)m

Now we have a displacement d2 = (-5, 0) m

After this, the position is:

(6, 1)m + (-5, 0)m = (6 -5, 1 + 0)m = (1, 1)m

After this, we have the final displacement d3 = (1, 6) m, so the final position will be:

(1, 1)m + (1, 6)m = (1 + 1, 1 + 6)m = (2, 7)m

Below you can see a rough sketch of the path that the student take, where he/she starts at point A.

DO U KNOW CARRYMINATI

Answers

Answer: Yes, I know



Explaination: Rough words and roaster

A exerted by an object on another is a force

Answers

hope this helps

Answer:

If a person is pushing a desk across the room, then there is an applied force acting upon the object. The applied force is the force exerted on the desk by the person.

A car accelerates for 10 seconds. During this time, the angular
velocity of the wheels of the car increases from 10 rad/s to 25 rad/s.
What is the angular acceleration of the wheels during this time?
e

Answers

Answer:

the angular acceleration of the car is 1.5 rad/s²

Explanation:

Given;

initial angular velocity, [tex]\omega_i[/tex] = 10 rad/s

final angular velocity, [tex]\omega_f[/tex] = 25 rad/s

time of motion, t = 10 s

The angular acceleration of the car is calculated as follows;

[tex]a_r = \frac{\omega_f - \omega_i }{t} \\\\a_r = \frac{25-10}{10} = 1.5 \ rad/s^2[/tex]

Therefore, the angular acceleration of the car is 1.5 rad/s²

Donald claims that he will always have more momentum than Jason because Donald has twice the mass of
Jason. Is Donald correct? Why or why not?

Answers

Answer:he is correct

Explanation: when Donald runs all his weight is moving at the same speed so when all that weight hits a stopped object the weight will transfer its force into that object Kinda like how a truck hits a car

what is time write its SI unit​

Answers

The SI unit of time is second

Questlon 20 of 20 Which change to an object would quadruple its kinetic energy?



A. Reducing its mass to one-half of its original value
B. Increasing its velocitato twice its original value
C. Reducing its velocity to one-half of its original value O
D. Increasing its mass to twice its original value ​

Answers

Answer:Whenever the velocity of the object increases or usually doubles, the kinetic energy will be quadrupled. If in case you are not familiar with kinetic energy, this is known as the type of energy that is in motion. The greater the mass of the object, the greater the total of the energy

Explanation:

Change to an object would quadruple its kinetic energy Increasing its velocity to twice its original value

What is kinetic energy?

In physics, the kinetic energy of an object is the energy that it possesses due to its motion. It is defined as the work needed to accelerate a body of a given mass from rest to its stated velocity. Having gained this energy during its acceleration, the body maintains this kinetic energy unless its speed changes. The same amount of work is done by the body when decelerating from its current speed to a state of rest. Formally, kinetic energy is any term in a system's Lagrangian which includes a derivative concerning time.

In classical mechanics, the kinetic energy of a non-rotating object of mass m traveling at a speed v is [tex]{\textstyle {\frac {1}{2}}mv^{2}}[/tex]

In relativistic mechanics, this is a good approximation only when v is much less than the speed of light.

The standard unit of kinetic energy is the joule, while the English unit of kinetic energy is the foot-pound.

Learn more about kinetic energy

https://brainly.com/question/12337396

#SPJ2

Which statement about momentum is false? a. Momentum is a force b. Momentum may be transferred c. Momentum is always conserved d. Momentum is a property only of moving objects

Answers

Answer:

So, option(a).

Explanation:

Momentum is the measure of motion of the object.

Momentum is given by the product of mass and the velocity of the object.

(a) It is false.

(b) It is true, momentum is transferred.

(c) It is true, As the momentum of the system is conserved when no external force is applied on the system.

(d) It is true, as the momentum is associated with the velocity of the object.

Brass is made by melting a mixture which contain 67% by mass of copper and 33% by mass of zinc if no change in volume.Calculate the density of brass​

Answers

Answer:

The density of brass is approximately 8,261.73 kg/m³

Explanation:

The percentage composition by mass of brass is given as follows;

The percentage by mass of copper = 67%

The percentage by mass of zinc = 33%

The density of copper, ρ₁ = 8.96 g/cm³

The density of zinc, ρ₂ = 7.133 g/cm³

Therefore, where we have, m = 100 g of brass, we have;

The mass of copper, m₁ = 67 g

The volume of copper, V₁ = m₁/ρ₁

∴ V₁ = 67 g/(8.96 g/cm³) ≈ 7.47767857 cm³

The volume of zinc, V₂ = m₂/ρ₂

∴ V₂ = 33 g/(7.133 g/cm³) ≈ 4.62638411 cm³

The volume of the brass, V = V₁ + V₂

V = 7.47767857 cm³ + 4.62638411 cm³ ≈ 12.104 cm³

The density of brass, ρ = m/V

∴ ρ = 100 g/(12.104 cm³) ≈ 8.26 g/cm³

The density of brass, ρ ≈ 8.26 g/cm³ = 8,261.73 kg/m³

Two wires made of the same material are connected in parallel
across the mains. The length and radius of one wire is two
times that of the other wire. If heat produced in the shorter wire
is 100 J, then the heat produced in the longer wire is:-
O A. 100 J
B. 180 J
O C. 2003
O D. 420 J​

Answers

Answer:

The heat produced in the longer wire is 200 J.

Explanation:

When the two wires are connected in parallel then the voltage across both the wires is same.

Let the length of the second wire is L and the radius of the second wire is R.

The length of the first ire is 2L and the radius of the first wire is 2R.

Heat produced in the second wire is 100 J.

Let the heat produced in the first wire is H'.

The resistance of the wire is proportional to length of the wire and inversely proportional to the area of the wire.

[tex]\frac{R_2}{R_1}=\frac{L\times \pi\times 4R^2}{2L\times \pi\times R^2}\\\\\frac{R_2}{R_1}=2[/tex]

The heat produced is given by

[tex]H=\frac{V^2}{R}\\\\\frac{H_1}{H_2}=\frac{R_2}{R_1}\\\\\frac{H_1}{100}=2\\\\H_1=200 J[/tex]

Temperature is the average amount of energy of motion in each particle of a substance. It is a measure of how loud or quiet a substance is. true of false

Answers

Answer:

fhddjdhdjndvdbhd honcho highchair suck Vicki

A stone is thrown vertically upward with an initial velocity of 40m/s. Taking g = 10 m/s^2 find the maximum height reach by the stone and what is the net displacement and distance covered by the stone.​

Answers

Explanation:

u=40

v=?

h=?

v²-u²=2gs

0²-40²=2×10×s

160=20s

s=160/20

=80m/s

total distance= upward distance ×downward distance

=80+80

=160m

total displacement=0 because u and v is the same.

Answer:

The maximum height is 80 m and the total distance covered by the stone is 160 m and the displacement is zero.

Explanation:

Final velocity v = 0

Initial velocity u = 40m/s

We know that,

Using equation of motion

[tex]v^{2} =u^{2} +2gh[/tex]

[tex]0-40^{2} =2[/tex] × [tex]10[/tex] × [tex]h[/tex]

The maximum height is:

[tex]h=80[/tex] [tex]m[/tex]

The  stone will reach at the top and will come down

Therefore, the total distance will be:

[tex]s=h_{1} +h_{2}[/tex]

[tex]s=80m-80m=160m[/tex]

The net displacement is:

[tex]D=h_{1} -h_{2}[/tex]

[tex]D=80m-80m=0[/tex]

Hence, The maximum height is 80 m and the total distance covered by the stone is 160 m and the displacement is zero.

hope this helps.....

a wooden block of mass 2kg is 20cm thick ,10cm wide and 30cm tall
A, calculate the minimum pressure
B, the maximum pressure the block exert on the surface​

Answers

Answer:

A.33333.3N/M2

B.100000N/M2

Explanation:

HAVE A NICE DAY ;]

The formula is  [tex]\frac{Newton}{Meter^{2} }[/tex]

10 Newton=1 kilogram

Have a nice day today

digestion that takes place in the mouth.​

Answers

Mechanical digestion..............

A car travelling at 30 ms' decelerates
to 10 m sin a distance of 20 m. What
is the deceleration of the motion, in m
s?

Answers

Answer:

V²=U² + 2as

Where V=Final Velocity

U=Initial Velocity

a=acceleration ( Deceleration in this case)

s=distance covered

10² = 30² + 2(a)(20)

100 = 900 + 40a

100 - 900 = 40a

-800=40a

a= -800/40

a= -20ms-².

Its negative because the car Decelerated.

If velocity is Hositive, which would most likely yield a negative acceleration?
O A final velocity that is faster than an initial velocity.
O A time that is less than a half hour.
O An initial velocity that is faster than a final velocity.
O A time that is greater than a half hour.

Answers

Answer:

A final velocity that is faster than an initial velocity.

Explanation:

The acceleration of an object is given by :

[tex]a=\dfrac{v-u}{t}[/tex]

Where

v is final velocity and u is initial velocity

t is time taken

If the final velocity is more than the initial velocity, the acceleration is positive. So, the correct option is (a).

Đặt điện áp U = 200 cos Omega t vào hai đầu đoạn mạch gồm biến trở R và tụ điện mắc nối tiếp điều chỉnh r để công suất tiêu thụ của đoạn mạch đạt cực đại Khi Điện áp tức thời hai đầu r = 100 V và đang tăng thì điện áp tức thời giữa hai đầu đoạn mạch là

Answers

Answer:

I don't understand the Question

which type of material is the best sound absorber?
a. glass
b. wood
c. concrete
d. carpet​

Answers

Explanation:

Glass is the best sound absorber material.

a
c) Determine the coefficient of static friction required to produce
net force of 6.0N?
Determine the coefficient of static friction to produce net force of 6.0N?

Answers

i think the data is not complete but that's according to me

Why is the time period of a charged particle in a uniform magnetic field independent of velocity??

Answers

Answer:

time period is independent of K.E of particle, it is inversely proportional to specific charge of the particle and it is independent of angle between velocity and magnetic field

Other Questions
The following program generates an error. Why? #include #include using namespace std; class Arcade { public: Arcade(); Arcade(string name, int r); void Print(); private: string arcName; int rating; }; Arcade:: Arcade() { arcName = "New"; rating = 1; } Arcade:: Arcade(string name, int r) { arcName = name; rating = r; } void Arcade:: Print() { cout 4.3.7 cm4.5 cm2.2 cmV= Summary of the story the beggar by anton chekhov Linda makes chocolate truffles . The recipe for 20 dark chocolate truffles is 1 cup cream 2 cups dark chocolate Complete the recipe for 40 dark truffles .............. is influenced by economic growth and development by technological change. Triangle Q R S is cut by line segment V W. Point V is the midpoint of side Q S and point W is the midpoint of side R S. The length of Q R is 3 a + 6, the length of V W is 2 a minus 2, and the length of V S is 2 a.If V is the midpoint of Line segment Q S and W is the midpoint of Line segment R S, then what is VS?4 units8 units10 units20 units Jackson decides to distribute buttons to advertise the no-smoking campaign. He plans to make 700 buttons. Large buttons cost 75 cents each and small buttons cost 35 cents each. He has $417 to spend on the buttons. At 4 p.m., the temperature started to change drastically. Each hour, for three hours the temperature decreased by 7FWhich expression best shows another way to write the product for the temperature change?07-7-7(-1)(7+7+7) Some foods undergo sterilization in order to kill bacteria living in and on thefood. What type of radiation is used in sterilization to kill bacteria?A. GammaB. AlphaC. BetaD. Fission Which of the following countries was created as a result of the American Revolution? A bicyclist rides 2.93 km due east, while the resistive force from the air has a magnitude of 8.65 N and points due west. The rider then turns around and rides 2.93 km due west, back to her starting point. The resistive force from the air on the return trip has a magnitude of 8.65 N and points due east. Find the work done by the resistive force during the round trip. Number Type your answer here Units Choose your answer here In the past year Diane watched 22 movies that she thought were very good. She watched 40 movies over the whole year. Of the movies she watched, what percentage did she rate as very good? Does a computer have internal organization In a paragraph, describe in detail a practical real-world example of where you would implement a singly-linked list and why a singly-linked list is best for this use. Do not list the properties of a singly-linked list....Rather identify a real-world example or application. If you identify this through research, cite your source. A recent news story reported that the Organization of Petroleum Exporting Countries is expected to decrease the supply of oil next summer. Summer is traditionally a time of increased demand for oil because of vacation travel. What would be the combined effect of these two events on the summer market for gasoline? a) an unpredictable change in the price and a decrease in the quantity b) an unpredictable change in both the price and the quantity c) an increase in the price and an unpredictable change in the quantity d) an increase in the price and the quantity I need help with this math problem please Find the area of this circle. Use 3 for pi. A = pi r2. radius is 15 ft so answer is [?] ft? Jason leaves Detroit at 4:00 PM and drives at a constant speed west along I-94. He passes Ann Arbor, 40 mi from Detroit, at 4:40 PM. (a) Express the distance d traveled in terms of the time t (in hours) elapsed. d(t) Which words best complete this sentence?He realized he was in a(n) _________ when he saw that the road was completely _________ with snow.A. nucleus, maneuveredB. descent, perpendicularC. predicament, envelopedD. ascent, precipitated A company needs to hire a new director of advertising. It has decided to try to hire person A or B, who are assistant advertising directors for its major competitor. To decide between A or B, the company finds that A is in charge of 2/3 of the total accounts while Peron B is in charge of only 1/3 of the accounts. Also, As campaigns have satisfactory results 3 out of 4 times, while Bs campaigns has satisfactory results only 2 out of 5 times. Suppose one of the competitors ad campaigns is selected randomly a) Draw the tree diagram of the info above b) Find the probability that A is in charge of the selected campaign and that is produces satisfactory results c) Find the probability that the selected campaign is satisfactory